A Very Illegal Position












20














It's been a while since I've done a chess puzzle, so here's one!



This position is illegal in quite a lot of ways:



enter image description here



It's illegal in at least 10 ways. The first one to find all of the ways this position is illegal (it's 10 or more, I know exactly how many) gets the check and +1 from me.



Good luck and happy puzzling!



Updated: I am considering, for the sake of this question, a way that this position is illegal to be any part of the position that could not have occurred from the starting position, even if it requires other illegal pieces to be that way.










share|improve this question
























  • i assume you mean for us to find the illegal ways and not the legal ways
    – AHKieran
    yesterday






  • 1




    @AHKieran fixed that little typo :P
    – Excited Raichu
    yesterday






  • 1




    It might help if you had a more specific definition of what a "way the position is illegal" consists of. I think I'm using a different definition, which is why I can only find 8.
    – isaacg
    22 hours ago










  • Obviously there's no way for the queen to still be on the board.
    – Steve
    3 hours ago
















20














It's been a while since I've done a chess puzzle, so here's one!



This position is illegal in quite a lot of ways:



enter image description here



It's illegal in at least 10 ways. The first one to find all of the ways this position is illegal (it's 10 or more, I know exactly how many) gets the check and +1 from me.



Good luck and happy puzzling!



Updated: I am considering, for the sake of this question, a way that this position is illegal to be any part of the position that could not have occurred from the starting position, even if it requires other illegal pieces to be that way.










share|improve this question
























  • i assume you mean for us to find the illegal ways and not the legal ways
    – AHKieran
    yesterday






  • 1




    @AHKieran fixed that little typo :P
    – Excited Raichu
    yesterday






  • 1




    It might help if you had a more specific definition of what a "way the position is illegal" consists of. I think I'm using a different definition, which is why I can only find 8.
    – isaacg
    22 hours ago










  • Obviously there's no way for the queen to still be on the board.
    – Steve
    3 hours ago














20












20








20


4





It's been a while since I've done a chess puzzle, so here's one!



This position is illegal in quite a lot of ways:



enter image description here



It's illegal in at least 10 ways. The first one to find all of the ways this position is illegal (it's 10 or more, I know exactly how many) gets the check and +1 from me.



Good luck and happy puzzling!



Updated: I am considering, for the sake of this question, a way that this position is illegal to be any part of the position that could not have occurred from the starting position, even if it requires other illegal pieces to be that way.










share|improve this question















It's been a while since I've done a chess puzzle, so here's one!



This position is illegal in quite a lot of ways:



enter image description here



It's illegal in at least 10 ways. The first one to find all of the ways this position is illegal (it's 10 or more, I know exactly how many) gets the check and +1 from me.



Good luck and happy puzzling!



Updated: I am considering, for the sake of this question, a way that this position is illegal to be any part of the position that could not have occurred from the starting position, even if it requires other illegal pieces to be that way.







chess






share|improve this question















share|improve this question













share|improve this question




share|improve this question








edited 10 hours ago

























asked yesterday









Excited Raichu

5,9632965




5,9632965












  • i assume you mean for us to find the illegal ways and not the legal ways
    – AHKieran
    yesterday






  • 1




    @AHKieran fixed that little typo :P
    – Excited Raichu
    yesterday






  • 1




    It might help if you had a more specific definition of what a "way the position is illegal" consists of. I think I'm using a different definition, which is why I can only find 8.
    – isaacg
    22 hours ago










  • Obviously there's no way for the queen to still be on the board.
    – Steve
    3 hours ago


















  • i assume you mean for us to find the illegal ways and not the legal ways
    – AHKieran
    yesterday






  • 1




    @AHKieran fixed that little typo :P
    – Excited Raichu
    yesterday






  • 1




    It might help if you had a more specific definition of what a "way the position is illegal" consists of. I think I'm using a different definition, which is why I can only find 8.
    – isaacg
    22 hours ago










  • Obviously there's no way for the queen to still be on the board.
    – Steve
    3 hours ago
















i assume you mean for us to find the illegal ways and not the legal ways
– AHKieran
yesterday




i assume you mean for us to find the illegal ways and not the legal ways
– AHKieran
yesterday




1




1




@AHKieran fixed that little typo :P
– Excited Raichu
yesterday




@AHKieran fixed that little typo :P
– Excited Raichu
yesterday




1




1




It might help if you had a more specific definition of what a "way the position is illegal" consists of. I think I'm using a different definition, which is why I can only find 8.
– isaacg
22 hours ago




It might help if you had a more specific definition of what a "way the position is illegal" consists of. I think I'm using a different definition, which is why I can only find 8.
– isaacg
22 hours ago












Obviously there's no way for the queen to still be on the board.
– Steve
3 hours ago




Obviously there's no way for the queen to still be on the board.
– Steve
3 hours ago










8 Answers
8






active

oldest

votes


















11














1.




Both kings are in check.




2.




The black king is in check by both the Rc3 and Bg4. Double-check is possible in certain cases, by either a discovered check or an en-passant capture, but neither can have happened here.




3.




The white bishop on b1 can only arrive there via a2 and c2, and there are white pawns on a2 and c2 which can't have moved.




4.




Black has nine pawns.




5.




White has two light-squared bishops; one must have been promoted but White has still eight pawns.




6.




White's pawns seem to have captured three times (b-pawn to d5, d-pawn to e3), but Black's only missing piece is a single knight. Also, the f7 pawn can only get there via captures (unless the black f6 pawn somehow dropped out of thin air, see 4.)




7.




There's no way for the black rook which started on h8 to escape (to either b6 or a1).






I'm not sure I can find more; for example,




it will be hard for the Ra1 to reach that square, but once you assume the Bb1 materialized out of thin air (see 3.), it's not a problem anymore.




Also,




the black pawns are 7 columns 'away' from their home squares, but if you assume the g3 has been dropped there, only 4 captures are required to account for the a- and b-pawns, and there are four white pieces missing (two knights, a rook and the queen).




A pitfall:




the white king can be put in check this way; Black's last move could have been Nh4+ (discovered check)




Note that




some of these irregularities could happen during a game of bughouse.







share|improve this answer























  • rook h8 could have been captured by a white knight, though
    – Bass
    yesterday






  • 1




    True, but that would require another promoted black pawn.
    – Glorfindel
    yesterday










  • Note that in a game of antichess bughouse, this is probably a valid position :)
    – Hosch250
    7 hours ago










  • On #2, it might be worth elaborating. Since, in real games, it's possible to have a double check when one check is revealed by the second checking piece. But in this particular case, there's no way for the rook or the bishop to have been blocking the other before the most recent move.
    – Shufflepants
    5 hours ago






  • 1




    @Shufflepants Players can't make moves that put their own king in check, so I don't think you can have a double check regardless.
    – W W
    3 hours ago



















7














Here's my 10 (I didn't look at anyone else's answers, honest)



1.




Nine black pawns.




2.




Black's h8 rook could not have left rank 8, yet black has two rooks, entailing a promotion from a tenth black pawn!




3.




Black pawns have made at least five captures, e.g. cxbxa3 and dxexfxg3, but White is missing only four units (queen, rook and two knights)




4.




Black's check with the bishop on f3 had to be a capture -- yet another capture by Black, and this capture, too, is invalid for the same reason as given in 3.




5.




White pawns have made at least three captures towards the kingside, e.g. bxcxd5 and dxe3. Moreover, two captures fxexf or fxgxf are needed to put a white pawn on f7. However, even if we remove a black pawn, black is missing only one unit (a rook).




6.




White's check with the bishop on g4 had to be a capture -- yet another capture by White, and this capture, too, is invalid for the same reason as given in 5.




7.




White's bishop on g4 and rook on c3 are giving an impossible double check.




8.




4. and 6. mean both kings are in check at the same time.




9.




White bishop on b1, though it wasn't born there and couldn't have moved there.




10.




White has two bishops on white squares but has eight pawns and thus has no promoted pieces.







share|improve this answer





















  • #4 is incorrect: Black last move could be Ng2-h4.
    – Evargalo
    14 hours ago










  • @Evargalo Oh yes, fair point.
    – Rosie F
    14 hours ago










  • #5 Black has both rooks, missing a knight.
    – Separatrix
    10 hours ago



















5















1. bishop in g4 in check

2. rook in c3 in check

3. bishop in f3 in check

4. rook in a1 cant get there

5. 9 black pawns

6. pawn in a3 cant get there

7. pawn in g3 cant get there

8. pawn in f7 cant get there

9. 2 white bishop on white squares

10. pawn in e3 cant get there

11. bishop in b1 cant get there







share|improve this answer



















  • 1




    Your #4 and #12 are the same. The A3 pawn could have come from the C column by capturing twice, same goes for G3 from the D column, although since black is only missing 2 pieces, one of these must be illegal.
    – Nuclear Wang
    yesterday



















5














I'm not sure how you're counting, but here's my answer. Possibly an incomplete answer.



1.




Both kings are in check.




2.




The black king is in check from two pieces.




3.




There are 9 black pawns. Removing the one at g3 makes the pawns at a3 and b2 valid via capturing white officers.




4.




There are not enough captured black pieces for the white pawns to be in those positions. The f7 pawn in particular is in a suspicious location, though this pawn configuration is possible if there were enough black officers to capture.




5.




The white bishop on b1 is in an impossible location.




6.




The white bishops are both on white squares, but all of the pawns are present.




7.




The black rook at a1 is in an impossible location... Sort of. It would be possible via promotion, but there already too many black pawns for that to be possible.




8.




There should be a black rook in the upper-right corner because there's no way it could get out with the bishop at f8.




That's all I see. I might be lumping together multiple items by your count into one.






share|improve this answer








New contributor




Beefster is a new contributor to this site. Take care in asking for clarification, commenting, and answering.
Check out our Code of Conduct.


























    4
















    1. Black has 9 pawns.




    2.




    Both kings are in check.




    3.




    The black king is in double check. This can only happen if one of the checking pieces shielded the opponent king from the other prior to the check. This is not possible with the took on c3 and the bishop on h4.




    4.




    The white bishop on b1 could never have reached this field with the white pawns on a2, c2.




    5.




    White has two bishops on white fields. This can only happen after a pawn has been promoted. White however still has their complete set of 8 pawns.




    6.




    With white pawns on the e and g files and a black pawn on f6, the white pawn on f7 can reach this field only with at least 2 captures. However, black has lost only 1 piece and all pawns (in fact even one surplus pawn, he. item 1), thus never had a promoted piece.




    7.




    White's pawn structure implies at least 5 captures (bxc, cxd; dxe; fxe, exf) but black has lost a single piece only and no promotion so far (cf. item 6)




    8.




    With black pawns on e7,f6,g7,h7 and a black bishop on f8, the black rook initially on h8 could only have reached the field g8.




    9.




    The black king is in check so white moved last. Since the white bishop on h4 can only move along the diagonal to c8 the black king must have been in check before the move; or black put itself into check by illegally moving a pinned piece; or white moved a different piece to open the diagonal, but white does not have a piece that could have moved this way [not sure if this counts though as the checking situation is impossible in the first place, cf. items 2 & 3]




    10.




    Black is in an 'impossible' double check and puts white in check (cf. items 2 & 3). That can only happen if black doesn't move out of a check, thus playing illegally.







    share|improve this answer





























      3














      I only managed to find 10 so far, here it is:




      1. White bishop in B1. It's not possible in a normal game.

      2. Two white bishops in the white square, while still having 8 pawns.

      3. Black Rook in A1. It's not possible in a normal game.

      4. Black Rook going out from H8 while Bishop hasn't moved from F8.

      5. White Rook missing from A1. Stolen and replaced by black rook? :P

      6. Black pawn totaling 9. It was supposed to be only 8 in a game.

      7. White king under check, while black king also in check.

      8. Black king under two different check, from Rook in C3 and Bishop on G4

      9. White pawn structure. It just simply weird to have a pawn on F7, D5 while also having a pawn on E3, while black only missing a knight.

      10. Both kings is under check, is simply impossible.







      share|improve this answer





















      • Black rook in A1 is possible (in isolation) as a promoted pawn having captured the White rook normally there. #7 and #10 are duplicates.
        – Separatrix
        9 hours ago












      • @Separatrix: Black has 9 pawns (in itself illegal), however, so there has not been a promotion. Or, if there was, black has been playing with at least 10 pawns at some moment in time, adding another illegal pawn (unless surplus pawns count as a single rule violation w/o regard to their number).
        – collapsar
        4 hours ago



















      3














      Definitely illegal



      1.




      Black has 9 pawns




      2.




      Rook a1 could be legal in isolation as a promoted pawn, but in light of #1, I'm declaring it illegal




      3.




      Bishop b1 cannot get there




      4.




      Both white bishops are on white, white has 8 pawns, one bishop is illegal




      5.




      Both kings are in check




      6.




      Black king is in illegal double check, could not be discovered by legal movement of either offending piece




      7.




      Black rook starting h8 could not have legally escaped, due to number of remaining black pawns, absence of rook considered illegal




      8.




      White pawns are at least 3 places off home columns, black still has 16 pieces, though missing a knight, at least two illegal places off home




      9.




      Pawn f7 is illegal, cannot have captured the required 2 pieces to return to home column behind black pawn as black not missing enough pieces




      10.




      Even assuming pawn g3 to be the interloper, black pawns are a minimum of 4 places off home column, white is missing 4 pieces, but rook a1 could only have been captured by a knight or queen, at least one illegal place off home




      Not illegal but might look it



      1.




      Bishop f3 check could be discovered by movement of knight h4 from g2, otherwise it's really hard to get into that position





      1. This ties into illegal #10



      Assuming bishop b1 to have been placed there at the start of the game, white rook starting a1 could not have legally escaped, but could have been captured by a knight or queen. Black bishop could not have been used.







      share|improve this answer































        2















        Bb1, no legal way to reach there
        Ra1, no legal way to reach there, due to Bb1
        Kh1, in check, other King in check
        Kc8, in double check, other King in check
        f7, no legal way to reach there
        Bf8, no legal way to reach there
        2 white bishops, with 8 pawns. Does this count as two illegals?







        share|improve this answer








        New contributor




        Kashyap is a new contributor to this site. Take care in asking for clarification, commenting, and answering.
        Check out our Code of Conduct.


















          Your Answer





          StackExchange.ifUsing("editor", function () {
          return StackExchange.using("mathjaxEditing", function () {
          StackExchange.MarkdownEditor.creationCallbacks.add(function (editor, postfix) {
          StackExchange.mathjaxEditing.prepareWmdForMathJax(editor, postfix, [["$", "$"], ["\\(","\\)"]]);
          });
          });
          }, "mathjax-editing");

          StackExchange.ready(function() {
          var channelOptions = {
          tags: "".split(" "),
          id: "559"
          };
          initTagRenderer("".split(" "), "".split(" "), channelOptions);

          StackExchange.using("externalEditor", function() {
          // Have to fire editor after snippets, if snippets enabled
          if (StackExchange.settings.snippets.snippetsEnabled) {
          StackExchange.using("snippets", function() {
          createEditor();
          });
          }
          else {
          createEditor();
          }
          });

          function createEditor() {
          StackExchange.prepareEditor({
          heartbeatType: 'answer',
          autoActivateHeartbeat: false,
          convertImagesToLinks: false,
          noModals: true,
          showLowRepImageUploadWarning: true,
          reputationToPostImages: null,
          bindNavPrevention: true,
          postfix: "",
          imageUploader: {
          brandingHtml: "Powered by u003ca class="icon-imgur-white" href="https://imgur.com/"u003eu003c/au003e",
          contentPolicyHtml: "User contributions licensed under u003ca href="https://creativecommons.org/licenses/by-sa/3.0/"u003ecc by-sa 3.0 with attribution requiredu003c/au003e u003ca href="https://stackoverflow.com/legal/content-policy"u003e(content policy)u003c/au003e",
          allowUrls: true
          },
          noCode: true, onDemand: true,
          discardSelector: ".discard-answer"
          ,immediatelyShowMarkdownHelp:true
          });


          }
          });














          draft saved

          draft discarded


















          StackExchange.ready(
          function () {
          StackExchange.openid.initPostLogin('.new-post-login', 'https%3a%2f%2fpuzzling.stackexchange.com%2fquestions%2f78081%2fa-very-illegal-position%23new-answer', 'question_page');
          }
          );

          Post as a guest















          Required, but never shown

























          8 Answers
          8






          active

          oldest

          votes








          8 Answers
          8






          active

          oldest

          votes









          active

          oldest

          votes






          active

          oldest

          votes









          11














          1.




          Both kings are in check.




          2.




          The black king is in check by both the Rc3 and Bg4. Double-check is possible in certain cases, by either a discovered check or an en-passant capture, but neither can have happened here.




          3.




          The white bishop on b1 can only arrive there via a2 and c2, and there are white pawns on a2 and c2 which can't have moved.




          4.




          Black has nine pawns.




          5.




          White has two light-squared bishops; one must have been promoted but White has still eight pawns.




          6.




          White's pawns seem to have captured three times (b-pawn to d5, d-pawn to e3), but Black's only missing piece is a single knight. Also, the f7 pawn can only get there via captures (unless the black f6 pawn somehow dropped out of thin air, see 4.)




          7.




          There's no way for the black rook which started on h8 to escape (to either b6 or a1).






          I'm not sure I can find more; for example,




          it will be hard for the Ra1 to reach that square, but once you assume the Bb1 materialized out of thin air (see 3.), it's not a problem anymore.




          Also,




          the black pawns are 7 columns 'away' from their home squares, but if you assume the g3 has been dropped there, only 4 captures are required to account for the a- and b-pawns, and there are four white pieces missing (two knights, a rook and the queen).




          A pitfall:




          the white king can be put in check this way; Black's last move could have been Nh4+ (discovered check)




          Note that




          some of these irregularities could happen during a game of bughouse.







          share|improve this answer























          • rook h8 could have been captured by a white knight, though
            – Bass
            yesterday






          • 1




            True, but that would require another promoted black pawn.
            – Glorfindel
            yesterday










          • Note that in a game of antichess bughouse, this is probably a valid position :)
            – Hosch250
            7 hours ago










          • On #2, it might be worth elaborating. Since, in real games, it's possible to have a double check when one check is revealed by the second checking piece. But in this particular case, there's no way for the rook or the bishop to have been blocking the other before the most recent move.
            – Shufflepants
            5 hours ago






          • 1




            @Shufflepants Players can't make moves that put their own king in check, so I don't think you can have a double check regardless.
            – W W
            3 hours ago
















          11














          1.




          Both kings are in check.




          2.




          The black king is in check by both the Rc3 and Bg4. Double-check is possible in certain cases, by either a discovered check or an en-passant capture, but neither can have happened here.




          3.




          The white bishop on b1 can only arrive there via a2 and c2, and there are white pawns on a2 and c2 which can't have moved.




          4.




          Black has nine pawns.




          5.




          White has two light-squared bishops; one must have been promoted but White has still eight pawns.




          6.




          White's pawns seem to have captured three times (b-pawn to d5, d-pawn to e3), but Black's only missing piece is a single knight. Also, the f7 pawn can only get there via captures (unless the black f6 pawn somehow dropped out of thin air, see 4.)




          7.




          There's no way for the black rook which started on h8 to escape (to either b6 or a1).






          I'm not sure I can find more; for example,




          it will be hard for the Ra1 to reach that square, but once you assume the Bb1 materialized out of thin air (see 3.), it's not a problem anymore.




          Also,




          the black pawns are 7 columns 'away' from their home squares, but if you assume the g3 has been dropped there, only 4 captures are required to account for the a- and b-pawns, and there are four white pieces missing (two knights, a rook and the queen).




          A pitfall:




          the white king can be put in check this way; Black's last move could have been Nh4+ (discovered check)




          Note that




          some of these irregularities could happen during a game of bughouse.







          share|improve this answer























          • rook h8 could have been captured by a white knight, though
            – Bass
            yesterday






          • 1




            True, but that would require another promoted black pawn.
            – Glorfindel
            yesterday










          • Note that in a game of antichess bughouse, this is probably a valid position :)
            – Hosch250
            7 hours ago










          • On #2, it might be worth elaborating. Since, in real games, it's possible to have a double check when one check is revealed by the second checking piece. But in this particular case, there's no way for the rook or the bishop to have been blocking the other before the most recent move.
            – Shufflepants
            5 hours ago






          • 1




            @Shufflepants Players can't make moves that put their own king in check, so I don't think you can have a double check regardless.
            – W W
            3 hours ago














          11












          11








          11






          1.




          Both kings are in check.




          2.




          The black king is in check by both the Rc3 and Bg4. Double-check is possible in certain cases, by either a discovered check or an en-passant capture, but neither can have happened here.




          3.




          The white bishop on b1 can only arrive there via a2 and c2, and there are white pawns on a2 and c2 which can't have moved.




          4.




          Black has nine pawns.




          5.




          White has two light-squared bishops; one must have been promoted but White has still eight pawns.




          6.




          White's pawns seem to have captured three times (b-pawn to d5, d-pawn to e3), but Black's only missing piece is a single knight. Also, the f7 pawn can only get there via captures (unless the black f6 pawn somehow dropped out of thin air, see 4.)




          7.




          There's no way for the black rook which started on h8 to escape (to either b6 or a1).






          I'm not sure I can find more; for example,




          it will be hard for the Ra1 to reach that square, but once you assume the Bb1 materialized out of thin air (see 3.), it's not a problem anymore.




          Also,




          the black pawns are 7 columns 'away' from their home squares, but if you assume the g3 has been dropped there, only 4 captures are required to account for the a- and b-pawns, and there are four white pieces missing (two knights, a rook and the queen).




          A pitfall:




          the white king can be put in check this way; Black's last move could have been Nh4+ (discovered check)




          Note that




          some of these irregularities could happen during a game of bughouse.







          share|improve this answer














          1.




          Both kings are in check.




          2.




          The black king is in check by both the Rc3 and Bg4. Double-check is possible in certain cases, by either a discovered check or an en-passant capture, but neither can have happened here.




          3.




          The white bishop on b1 can only arrive there via a2 and c2, and there are white pawns on a2 and c2 which can't have moved.




          4.




          Black has nine pawns.




          5.




          White has two light-squared bishops; one must have been promoted but White has still eight pawns.




          6.




          White's pawns seem to have captured three times (b-pawn to d5, d-pawn to e3), but Black's only missing piece is a single knight. Also, the f7 pawn can only get there via captures (unless the black f6 pawn somehow dropped out of thin air, see 4.)




          7.




          There's no way for the black rook which started on h8 to escape (to either b6 or a1).






          I'm not sure I can find more; for example,




          it will be hard for the Ra1 to reach that square, but once you assume the Bb1 materialized out of thin air (see 3.), it's not a problem anymore.




          Also,




          the black pawns are 7 columns 'away' from their home squares, but if you assume the g3 has been dropped there, only 4 captures are required to account for the a- and b-pawns, and there are four white pieces missing (two knights, a rook and the queen).




          A pitfall:




          the white king can be put in check this way; Black's last move could have been Nh4+ (discovered check)




          Note that




          some of these irregularities could happen during a game of bughouse.








          share|improve this answer














          share|improve this answer



          share|improve this answer








          edited 5 hours ago

























          answered yesterday









          Glorfindel

          13.4k34983




          13.4k34983












          • rook h8 could have been captured by a white knight, though
            – Bass
            yesterday






          • 1




            True, but that would require another promoted black pawn.
            – Glorfindel
            yesterday










          • Note that in a game of antichess bughouse, this is probably a valid position :)
            – Hosch250
            7 hours ago










          • On #2, it might be worth elaborating. Since, in real games, it's possible to have a double check when one check is revealed by the second checking piece. But in this particular case, there's no way for the rook or the bishop to have been blocking the other before the most recent move.
            – Shufflepants
            5 hours ago






          • 1




            @Shufflepants Players can't make moves that put their own king in check, so I don't think you can have a double check regardless.
            – W W
            3 hours ago


















          • rook h8 could have been captured by a white knight, though
            – Bass
            yesterday






          • 1




            True, but that would require another promoted black pawn.
            – Glorfindel
            yesterday










          • Note that in a game of antichess bughouse, this is probably a valid position :)
            – Hosch250
            7 hours ago










          • On #2, it might be worth elaborating. Since, in real games, it's possible to have a double check when one check is revealed by the second checking piece. But in this particular case, there's no way for the rook or the bishop to have been blocking the other before the most recent move.
            – Shufflepants
            5 hours ago






          • 1




            @Shufflepants Players can't make moves that put their own king in check, so I don't think you can have a double check regardless.
            – W W
            3 hours ago
















          rook h8 could have been captured by a white knight, though
          – Bass
          yesterday




          rook h8 could have been captured by a white knight, though
          – Bass
          yesterday




          1




          1




          True, but that would require another promoted black pawn.
          – Glorfindel
          yesterday




          True, but that would require another promoted black pawn.
          – Glorfindel
          yesterday












          Note that in a game of antichess bughouse, this is probably a valid position :)
          – Hosch250
          7 hours ago




          Note that in a game of antichess bughouse, this is probably a valid position :)
          – Hosch250
          7 hours ago












          On #2, it might be worth elaborating. Since, in real games, it's possible to have a double check when one check is revealed by the second checking piece. But in this particular case, there's no way for the rook or the bishop to have been blocking the other before the most recent move.
          – Shufflepants
          5 hours ago




          On #2, it might be worth elaborating. Since, in real games, it's possible to have a double check when one check is revealed by the second checking piece. But in this particular case, there's no way for the rook or the bishop to have been blocking the other before the most recent move.
          – Shufflepants
          5 hours ago




          1




          1




          @Shufflepants Players can't make moves that put their own king in check, so I don't think you can have a double check regardless.
          – W W
          3 hours ago




          @Shufflepants Players can't make moves that put their own king in check, so I don't think you can have a double check regardless.
          – W W
          3 hours ago











          7














          Here's my 10 (I didn't look at anyone else's answers, honest)



          1.




          Nine black pawns.




          2.




          Black's h8 rook could not have left rank 8, yet black has two rooks, entailing a promotion from a tenth black pawn!




          3.




          Black pawns have made at least five captures, e.g. cxbxa3 and dxexfxg3, but White is missing only four units (queen, rook and two knights)




          4.




          Black's check with the bishop on f3 had to be a capture -- yet another capture by Black, and this capture, too, is invalid for the same reason as given in 3.




          5.




          White pawns have made at least three captures towards the kingside, e.g. bxcxd5 and dxe3. Moreover, two captures fxexf or fxgxf are needed to put a white pawn on f7. However, even if we remove a black pawn, black is missing only one unit (a rook).




          6.




          White's check with the bishop on g4 had to be a capture -- yet another capture by White, and this capture, too, is invalid for the same reason as given in 5.




          7.




          White's bishop on g4 and rook on c3 are giving an impossible double check.




          8.




          4. and 6. mean both kings are in check at the same time.




          9.




          White bishop on b1, though it wasn't born there and couldn't have moved there.




          10.




          White has two bishops on white squares but has eight pawns and thus has no promoted pieces.







          share|improve this answer





















          • #4 is incorrect: Black last move could be Ng2-h4.
            – Evargalo
            14 hours ago










          • @Evargalo Oh yes, fair point.
            – Rosie F
            14 hours ago










          • #5 Black has both rooks, missing a knight.
            – Separatrix
            10 hours ago
















          7














          Here's my 10 (I didn't look at anyone else's answers, honest)



          1.




          Nine black pawns.




          2.




          Black's h8 rook could not have left rank 8, yet black has two rooks, entailing a promotion from a tenth black pawn!




          3.




          Black pawns have made at least five captures, e.g. cxbxa3 and dxexfxg3, but White is missing only four units (queen, rook and two knights)




          4.




          Black's check with the bishop on f3 had to be a capture -- yet another capture by Black, and this capture, too, is invalid for the same reason as given in 3.




          5.




          White pawns have made at least three captures towards the kingside, e.g. bxcxd5 and dxe3. Moreover, two captures fxexf or fxgxf are needed to put a white pawn on f7. However, even if we remove a black pawn, black is missing only one unit (a rook).




          6.




          White's check with the bishop on g4 had to be a capture -- yet another capture by White, and this capture, too, is invalid for the same reason as given in 5.




          7.




          White's bishop on g4 and rook on c3 are giving an impossible double check.




          8.




          4. and 6. mean both kings are in check at the same time.




          9.




          White bishop on b1, though it wasn't born there and couldn't have moved there.




          10.




          White has two bishops on white squares but has eight pawns and thus has no promoted pieces.







          share|improve this answer





















          • #4 is incorrect: Black last move could be Ng2-h4.
            – Evargalo
            14 hours ago










          • @Evargalo Oh yes, fair point.
            – Rosie F
            14 hours ago










          • #5 Black has both rooks, missing a knight.
            – Separatrix
            10 hours ago














          7












          7








          7






          Here's my 10 (I didn't look at anyone else's answers, honest)



          1.




          Nine black pawns.




          2.




          Black's h8 rook could not have left rank 8, yet black has two rooks, entailing a promotion from a tenth black pawn!




          3.




          Black pawns have made at least five captures, e.g. cxbxa3 and dxexfxg3, but White is missing only four units (queen, rook and two knights)




          4.




          Black's check with the bishop on f3 had to be a capture -- yet another capture by Black, and this capture, too, is invalid for the same reason as given in 3.




          5.




          White pawns have made at least three captures towards the kingside, e.g. bxcxd5 and dxe3. Moreover, two captures fxexf or fxgxf are needed to put a white pawn on f7. However, even if we remove a black pawn, black is missing only one unit (a rook).




          6.




          White's check with the bishop on g4 had to be a capture -- yet another capture by White, and this capture, too, is invalid for the same reason as given in 5.




          7.




          White's bishop on g4 and rook on c3 are giving an impossible double check.




          8.




          4. and 6. mean both kings are in check at the same time.




          9.




          White bishop on b1, though it wasn't born there and couldn't have moved there.




          10.




          White has two bishops on white squares but has eight pawns and thus has no promoted pieces.







          share|improve this answer












          Here's my 10 (I didn't look at anyone else's answers, honest)



          1.




          Nine black pawns.




          2.




          Black's h8 rook could not have left rank 8, yet black has two rooks, entailing a promotion from a tenth black pawn!




          3.




          Black pawns have made at least five captures, e.g. cxbxa3 and dxexfxg3, but White is missing only four units (queen, rook and two knights)




          4.




          Black's check with the bishop on f3 had to be a capture -- yet another capture by Black, and this capture, too, is invalid for the same reason as given in 3.




          5.




          White pawns have made at least three captures towards the kingside, e.g. bxcxd5 and dxe3. Moreover, two captures fxexf or fxgxf are needed to put a white pawn on f7. However, even if we remove a black pawn, black is missing only one unit (a rook).




          6.




          White's check with the bishop on g4 had to be a capture -- yet another capture by White, and this capture, too, is invalid for the same reason as given in 5.




          7.




          White's bishop on g4 and rook on c3 are giving an impossible double check.




          8.




          4. and 6. mean both kings are in check at the same time.




          9.




          White bishop on b1, though it wasn't born there and couldn't have moved there.




          10.




          White has two bishops on white squares but has eight pawns and thus has no promoted pieces.








          share|improve this answer












          share|improve this answer



          share|improve this answer










          answered yesterday









          Rosie F

          5,7182943




          5,7182943












          • #4 is incorrect: Black last move could be Ng2-h4.
            – Evargalo
            14 hours ago










          • @Evargalo Oh yes, fair point.
            – Rosie F
            14 hours ago










          • #5 Black has both rooks, missing a knight.
            – Separatrix
            10 hours ago


















          • #4 is incorrect: Black last move could be Ng2-h4.
            – Evargalo
            14 hours ago










          • @Evargalo Oh yes, fair point.
            – Rosie F
            14 hours ago










          • #5 Black has both rooks, missing a knight.
            – Separatrix
            10 hours ago
















          #4 is incorrect: Black last move could be Ng2-h4.
          – Evargalo
          14 hours ago




          #4 is incorrect: Black last move could be Ng2-h4.
          – Evargalo
          14 hours ago












          @Evargalo Oh yes, fair point.
          – Rosie F
          14 hours ago




          @Evargalo Oh yes, fair point.
          – Rosie F
          14 hours ago












          #5 Black has both rooks, missing a knight.
          – Separatrix
          10 hours ago




          #5 Black has both rooks, missing a knight.
          – Separatrix
          10 hours ago











          5















          1. bishop in g4 in check

          2. rook in c3 in check

          3. bishop in f3 in check

          4. rook in a1 cant get there

          5. 9 black pawns

          6. pawn in a3 cant get there

          7. pawn in g3 cant get there

          8. pawn in f7 cant get there

          9. 2 white bishop on white squares

          10. pawn in e3 cant get there

          11. bishop in b1 cant get there







          share|improve this answer



















          • 1




            Your #4 and #12 are the same. The A3 pawn could have come from the C column by capturing twice, same goes for G3 from the D column, although since black is only missing 2 pieces, one of these must be illegal.
            – Nuclear Wang
            yesterday
















          5















          1. bishop in g4 in check

          2. rook in c3 in check

          3. bishop in f3 in check

          4. rook in a1 cant get there

          5. 9 black pawns

          6. pawn in a3 cant get there

          7. pawn in g3 cant get there

          8. pawn in f7 cant get there

          9. 2 white bishop on white squares

          10. pawn in e3 cant get there

          11. bishop in b1 cant get there







          share|improve this answer



















          • 1




            Your #4 and #12 are the same. The A3 pawn could have come from the C column by capturing twice, same goes for G3 from the D column, although since black is only missing 2 pieces, one of these must be illegal.
            – Nuclear Wang
            yesterday














          5












          5








          5







          1. bishop in g4 in check

          2. rook in c3 in check

          3. bishop in f3 in check

          4. rook in a1 cant get there

          5. 9 black pawns

          6. pawn in a3 cant get there

          7. pawn in g3 cant get there

          8. pawn in f7 cant get there

          9. 2 white bishop on white squares

          10. pawn in e3 cant get there

          11. bishop in b1 cant get there







          share|improve this answer















          1. bishop in g4 in check

          2. rook in c3 in check

          3. bishop in f3 in check

          4. rook in a1 cant get there

          5. 9 black pawns

          6. pawn in a3 cant get there

          7. pawn in g3 cant get there

          8. pawn in f7 cant get there

          9. 2 white bishop on white squares

          10. pawn in e3 cant get there

          11. bishop in b1 cant get there








          share|improve this answer














          share|improve this answer



          share|improve this answer








          edited yesterday

























          answered yesterday









          pirate

          531115




          531115








          • 1




            Your #4 and #12 are the same. The A3 pawn could have come from the C column by capturing twice, same goes for G3 from the D column, although since black is only missing 2 pieces, one of these must be illegal.
            – Nuclear Wang
            yesterday














          • 1




            Your #4 and #12 are the same. The A3 pawn could have come from the C column by capturing twice, same goes for G3 from the D column, although since black is only missing 2 pieces, one of these must be illegal.
            – Nuclear Wang
            yesterday








          1




          1




          Your #4 and #12 are the same. The A3 pawn could have come from the C column by capturing twice, same goes for G3 from the D column, although since black is only missing 2 pieces, one of these must be illegal.
          – Nuclear Wang
          yesterday




          Your #4 and #12 are the same. The A3 pawn could have come from the C column by capturing twice, same goes for G3 from the D column, although since black is only missing 2 pieces, one of these must be illegal.
          – Nuclear Wang
          yesterday











          5














          I'm not sure how you're counting, but here's my answer. Possibly an incomplete answer.



          1.




          Both kings are in check.




          2.




          The black king is in check from two pieces.




          3.




          There are 9 black pawns. Removing the one at g3 makes the pawns at a3 and b2 valid via capturing white officers.




          4.




          There are not enough captured black pieces for the white pawns to be in those positions. The f7 pawn in particular is in a suspicious location, though this pawn configuration is possible if there were enough black officers to capture.




          5.




          The white bishop on b1 is in an impossible location.




          6.




          The white bishops are both on white squares, but all of the pawns are present.




          7.




          The black rook at a1 is in an impossible location... Sort of. It would be possible via promotion, but there already too many black pawns for that to be possible.




          8.




          There should be a black rook in the upper-right corner because there's no way it could get out with the bishop at f8.




          That's all I see. I might be lumping together multiple items by your count into one.






          share|improve this answer








          New contributor




          Beefster is a new contributor to this site. Take care in asking for clarification, commenting, and answering.
          Check out our Code of Conduct.























            5














            I'm not sure how you're counting, but here's my answer. Possibly an incomplete answer.



            1.




            Both kings are in check.




            2.




            The black king is in check from two pieces.




            3.




            There are 9 black pawns. Removing the one at g3 makes the pawns at a3 and b2 valid via capturing white officers.




            4.




            There are not enough captured black pieces for the white pawns to be in those positions. The f7 pawn in particular is in a suspicious location, though this pawn configuration is possible if there were enough black officers to capture.




            5.




            The white bishop on b1 is in an impossible location.




            6.




            The white bishops are both on white squares, but all of the pawns are present.




            7.




            The black rook at a1 is in an impossible location... Sort of. It would be possible via promotion, but there already too many black pawns for that to be possible.




            8.




            There should be a black rook in the upper-right corner because there's no way it could get out with the bishop at f8.




            That's all I see. I might be lumping together multiple items by your count into one.






            share|improve this answer








            New contributor




            Beefster is a new contributor to this site. Take care in asking for clarification, commenting, and answering.
            Check out our Code of Conduct.





















              5












              5








              5






              I'm not sure how you're counting, but here's my answer. Possibly an incomplete answer.



              1.




              Both kings are in check.




              2.




              The black king is in check from two pieces.




              3.




              There are 9 black pawns. Removing the one at g3 makes the pawns at a3 and b2 valid via capturing white officers.




              4.




              There are not enough captured black pieces for the white pawns to be in those positions. The f7 pawn in particular is in a suspicious location, though this pawn configuration is possible if there were enough black officers to capture.




              5.




              The white bishop on b1 is in an impossible location.




              6.




              The white bishops are both on white squares, but all of the pawns are present.




              7.




              The black rook at a1 is in an impossible location... Sort of. It would be possible via promotion, but there already too many black pawns for that to be possible.




              8.




              There should be a black rook in the upper-right corner because there's no way it could get out with the bishop at f8.




              That's all I see. I might be lumping together multiple items by your count into one.






              share|improve this answer








              New contributor




              Beefster is a new contributor to this site. Take care in asking for clarification, commenting, and answering.
              Check out our Code of Conduct.









              I'm not sure how you're counting, but here's my answer. Possibly an incomplete answer.



              1.




              Both kings are in check.




              2.




              The black king is in check from two pieces.




              3.




              There are 9 black pawns. Removing the one at g3 makes the pawns at a3 and b2 valid via capturing white officers.




              4.




              There are not enough captured black pieces for the white pawns to be in those positions. The f7 pawn in particular is in a suspicious location, though this pawn configuration is possible if there were enough black officers to capture.




              5.




              The white bishop on b1 is in an impossible location.




              6.




              The white bishops are both on white squares, but all of the pawns are present.




              7.




              The black rook at a1 is in an impossible location... Sort of. It would be possible via promotion, but there already too many black pawns for that to be possible.




              8.




              There should be a black rook in the upper-right corner because there's no way it could get out with the bishop at f8.




              That's all I see. I might be lumping together multiple items by your count into one.







              share|improve this answer








              New contributor




              Beefster is a new contributor to this site. Take care in asking for clarification, commenting, and answering.
              Check out our Code of Conduct.









              share|improve this answer



              share|improve this answer






              New contributor




              Beefster is a new contributor to this site. Take care in asking for clarification, commenting, and answering.
              Check out our Code of Conduct.









              answered yesterday









              Beefster

              1512




              1512




              New contributor




              Beefster is a new contributor to this site. Take care in asking for clarification, commenting, and answering.
              Check out our Code of Conduct.





              New contributor





              Beefster is a new contributor to this site. Take care in asking for clarification, commenting, and answering.
              Check out our Code of Conduct.






              Beefster is a new contributor to this site. Take care in asking for clarification, commenting, and answering.
              Check out our Code of Conduct.























                  4
















                  1. Black has 9 pawns.




                  2.




                  Both kings are in check.




                  3.




                  The black king is in double check. This can only happen if one of the checking pieces shielded the opponent king from the other prior to the check. This is not possible with the took on c3 and the bishop on h4.




                  4.




                  The white bishop on b1 could never have reached this field with the white pawns on a2, c2.




                  5.




                  White has two bishops on white fields. This can only happen after a pawn has been promoted. White however still has their complete set of 8 pawns.




                  6.




                  With white pawns on the e and g files and a black pawn on f6, the white pawn on f7 can reach this field only with at least 2 captures. However, black has lost only 1 piece and all pawns (in fact even one surplus pawn, he. item 1), thus never had a promoted piece.




                  7.




                  White's pawn structure implies at least 5 captures (bxc, cxd; dxe; fxe, exf) but black has lost a single piece only and no promotion so far (cf. item 6)




                  8.




                  With black pawns on e7,f6,g7,h7 and a black bishop on f8, the black rook initially on h8 could only have reached the field g8.




                  9.




                  The black king is in check so white moved last. Since the white bishop on h4 can only move along the diagonal to c8 the black king must have been in check before the move; or black put itself into check by illegally moving a pinned piece; or white moved a different piece to open the diagonal, but white does not have a piece that could have moved this way [not sure if this counts though as the checking situation is impossible in the first place, cf. items 2 & 3]




                  10.




                  Black is in an 'impossible' double check and puts white in check (cf. items 2 & 3). That can only happen if black doesn't move out of a check, thus playing illegally.







                  share|improve this answer


























                    4
















                    1. Black has 9 pawns.




                    2.




                    Both kings are in check.




                    3.




                    The black king is in double check. This can only happen if one of the checking pieces shielded the opponent king from the other prior to the check. This is not possible with the took on c3 and the bishop on h4.




                    4.




                    The white bishop on b1 could never have reached this field with the white pawns on a2, c2.




                    5.




                    White has two bishops on white fields. This can only happen after a pawn has been promoted. White however still has their complete set of 8 pawns.




                    6.




                    With white pawns on the e and g files and a black pawn on f6, the white pawn on f7 can reach this field only with at least 2 captures. However, black has lost only 1 piece and all pawns (in fact even one surplus pawn, he. item 1), thus never had a promoted piece.




                    7.




                    White's pawn structure implies at least 5 captures (bxc, cxd; dxe; fxe, exf) but black has lost a single piece only and no promotion so far (cf. item 6)




                    8.




                    With black pawns on e7,f6,g7,h7 and a black bishop on f8, the black rook initially on h8 could only have reached the field g8.




                    9.




                    The black king is in check so white moved last. Since the white bishop on h4 can only move along the diagonal to c8 the black king must have been in check before the move; or black put itself into check by illegally moving a pinned piece; or white moved a different piece to open the diagonal, but white does not have a piece that could have moved this way [not sure if this counts though as the checking situation is impossible in the first place, cf. items 2 & 3]




                    10.




                    Black is in an 'impossible' double check and puts white in check (cf. items 2 & 3). That can only happen if black doesn't move out of a check, thus playing illegally.







                    share|improve this answer
























                      4












                      4








                      4








                      1. Black has 9 pawns.




                      2.




                      Both kings are in check.




                      3.




                      The black king is in double check. This can only happen if one of the checking pieces shielded the opponent king from the other prior to the check. This is not possible with the took on c3 and the bishop on h4.




                      4.




                      The white bishop on b1 could never have reached this field with the white pawns on a2, c2.




                      5.




                      White has two bishops on white fields. This can only happen after a pawn has been promoted. White however still has their complete set of 8 pawns.




                      6.




                      With white pawns on the e and g files and a black pawn on f6, the white pawn on f7 can reach this field only with at least 2 captures. However, black has lost only 1 piece and all pawns (in fact even one surplus pawn, he. item 1), thus never had a promoted piece.




                      7.




                      White's pawn structure implies at least 5 captures (bxc, cxd; dxe; fxe, exf) but black has lost a single piece only and no promotion so far (cf. item 6)




                      8.




                      With black pawns on e7,f6,g7,h7 and a black bishop on f8, the black rook initially on h8 could only have reached the field g8.




                      9.




                      The black king is in check so white moved last. Since the white bishop on h4 can only move along the diagonal to c8 the black king must have been in check before the move; or black put itself into check by illegally moving a pinned piece; or white moved a different piece to open the diagonal, but white does not have a piece that could have moved this way [not sure if this counts though as the checking situation is impossible in the first place, cf. items 2 & 3]




                      10.




                      Black is in an 'impossible' double check and puts white in check (cf. items 2 & 3). That can only happen if black doesn't move out of a check, thus playing illegally.







                      share|improve this answer














                      1. Black has 9 pawns.




                      2.




                      Both kings are in check.




                      3.




                      The black king is in double check. This can only happen if one of the checking pieces shielded the opponent king from the other prior to the check. This is not possible with the took on c3 and the bishop on h4.




                      4.




                      The white bishop on b1 could never have reached this field with the white pawns on a2, c2.




                      5.




                      White has two bishops on white fields. This can only happen after a pawn has been promoted. White however still has their complete set of 8 pawns.




                      6.




                      With white pawns on the e and g files and a black pawn on f6, the white pawn on f7 can reach this field only with at least 2 captures. However, black has lost only 1 piece and all pawns (in fact even one surplus pawn, he. item 1), thus never had a promoted piece.




                      7.




                      White's pawn structure implies at least 5 captures (bxc, cxd; dxe; fxe, exf) but black has lost a single piece only and no promotion so far (cf. item 6)




                      8.




                      With black pawns on e7,f6,g7,h7 and a black bishop on f8, the black rook initially on h8 could only have reached the field g8.




                      9.




                      The black king is in check so white moved last. Since the white bishop on h4 can only move along the diagonal to c8 the black king must have been in check before the move; or black put itself into check by illegally moving a pinned piece; or white moved a different piece to open the diagonal, but white does not have a piece that could have moved this way [not sure if this counts though as the checking situation is impossible in the first place, cf. items 2 & 3]




                      10.




                      Black is in an 'impossible' double check and puts white in check (cf. items 2 & 3). That can only happen if black doesn't move out of a check, thus playing illegally.








                      share|improve this answer












                      share|improve this answer



                      share|improve this answer










                      answered 20 hours ago









                      collapsar

                      1494




                      1494























                          3














                          I only managed to find 10 so far, here it is:




                          1. White bishop in B1. It's not possible in a normal game.

                          2. Two white bishops in the white square, while still having 8 pawns.

                          3. Black Rook in A1. It's not possible in a normal game.

                          4. Black Rook going out from H8 while Bishop hasn't moved from F8.

                          5. White Rook missing from A1. Stolen and replaced by black rook? :P

                          6. Black pawn totaling 9. It was supposed to be only 8 in a game.

                          7. White king under check, while black king also in check.

                          8. Black king under two different check, from Rook in C3 and Bishop on G4

                          9. White pawn structure. It just simply weird to have a pawn on F7, D5 while also having a pawn on E3, while black only missing a knight.

                          10. Both kings is under check, is simply impossible.







                          share|improve this answer





















                          • Black rook in A1 is possible (in isolation) as a promoted pawn having captured the White rook normally there. #7 and #10 are duplicates.
                            – Separatrix
                            9 hours ago












                          • @Separatrix: Black has 9 pawns (in itself illegal), however, so there has not been a promotion. Or, if there was, black has been playing with at least 10 pawns at some moment in time, adding another illegal pawn (unless surplus pawns count as a single rule violation w/o regard to their number).
                            – collapsar
                            4 hours ago
















                          3














                          I only managed to find 10 so far, here it is:




                          1. White bishop in B1. It's not possible in a normal game.

                          2. Two white bishops in the white square, while still having 8 pawns.

                          3. Black Rook in A1. It's not possible in a normal game.

                          4. Black Rook going out from H8 while Bishop hasn't moved from F8.

                          5. White Rook missing from A1. Stolen and replaced by black rook? :P

                          6. Black pawn totaling 9. It was supposed to be only 8 in a game.

                          7. White king under check, while black king also in check.

                          8. Black king under two different check, from Rook in C3 and Bishop on G4

                          9. White pawn structure. It just simply weird to have a pawn on F7, D5 while also having a pawn on E3, while black only missing a knight.

                          10. Both kings is under check, is simply impossible.







                          share|improve this answer





















                          • Black rook in A1 is possible (in isolation) as a promoted pawn having captured the White rook normally there. #7 and #10 are duplicates.
                            – Separatrix
                            9 hours ago












                          • @Separatrix: Black has 9 pawns (in itself illegal), however, so there has not been a promotion. Or, if there was, black has been playing with at least 10 pawns at some moment in time, adding another illegal pawn (unless surplus pawns count as a single rule violation w/o regard to their number).
                            – collapsar
                            4 hours ago














                          3












                          3








                          3






                          I only managed to find 10 so far, here it is:




                          1. White bishop in B1. It's not possible in a normal game.

                          2. Two white bishops in the white square, while still having 8 pawns.

                          3. Black Rook in A1. It's not possible in a normal game.

                          4. Black Rook going out from H8 while Bishop hasn't moved from F8.

                          5. White Rook missing from A1. Stolen and replaced by black rook? :P

                          6. Black pawn totaling 9. It was supposed to be only 8 in a game.

                          7. White king under check, while black king also in check.

                          8. Black king under two different check, from Rook in C3 and Bishop on G4

                          9. White pawn structure. It just simply weird to have a pawn on F7, D5 while also having a pawn on E3, while black only missing a knight.

                          10. Both kings is under check, is simply impossible.







                          share|improve this answer












                          I only managed to find 10 so far, here it is:




                          1. White bishop in B1. It's not possible in a normal game.

                          2. Two white bishops in the white square, while still having 8 pawns.

                          3. Black Rook in A1. It's not possible in a normal game.

                          4. Black Rook going out from H8 while Bishop hasn't moved from F8.

                          5. White Rook missing from A1. Stolen and replaced by black rook? :P

                          6. Black pawn totaling 9. It was supposed to be only 8 in a game.

                          7. White king under check, while black king also in check.

                          8. Black king under two different check, from Rook in C3 and Bishop on G4

                          9. White pawn structure. It just simply weird to have a pawn on F7, D5 while also having a pawn on E3, while black only missing a knight.

                          10. Both kings is under check, is simply impossible.








                          share|improve this answer












                          share|improve this answer



                          share|improve this answer










                          answered 20 hours ago









                          Mukyuu

                          273112




                          273112












                          • Black rook in A1 is possible (in isolation) as a promoted pawn having captured the White rook normally there. #7 and #10 are duplicates.
                            – Separatrix
                            9 hours ago












                          • @Separatrix: Black has 9 pawns (in itself illegal), however, so there has not been a promotion. Or, if there was, black has been playing with at least 10 pawns at some moment in time, adding another illegal pawn (unless surplus pawns count as a single rule violation w/o regard to their number).
                            – collapsar
                            4 hours ago


















                          • Black rook in A1 is possible (in isolation) as a promoted pawn having captured the White rook normally there. #7 and #10 are duplicates.
                            – Separatrix
                            9 hours ago












                          • @Separatrix: Black has 9 pawns (in itself illegal), however, so there has not been a promotion. Or, if there was, black has been playing with at least 10 pawns at some moment in time, adding another illegal pawn (unless surplus pawns count as a single rule violation w/o regard to their number).
                            – collapsar
                            4 hours ago
















                          Black rook in A1 is possible (in isolation) as a promoted pawn having captured the White rook normally there. #7 and #10 are duplicates.
                          – Separatrix
                          9 hours ago






                          Black rook in A1 is possible (in isolation) as a promoted pawn having captured the White rook normally there. #7 and #10 are duplicates.
                          – Separatrix
                          9 hours ago














                          @Separatrix: Black has 9 pawns (in itself illegal), however, so there has not been a promotion. Or, if there was, black has been playing with at least 10 pawns at some moment in time, adding another illegal pawn (unless surplus pawns count as a single rule violation w/o regard to their number).
                          – collapsar
                          4 hours ago




                          @Separatrix: Black has 9 pawns (in itself illegal), however, so there has not been a promotion. Or, if there was, black has been playing with at least 10 pawns at some moment in time, adding another illegal pawn (unless surplus pawns count as a single rule violation w/o regard to their number).
                          – collapsar
                          4 hours ago











                          3














                          Definitely illegal



                          1.




                          Black has 9 pawns




                          2.




                          Rook a1 could be legal in isolation as a promoted pawn, but in light of #1, I'm declaring it illegal




                          3.




                          Bishop b1 cannot get there




                          4.




                          Both white bishops are on white, white has 8 pawns, one bishop is illegal




                          5.




                          Both kings are in check




                          6.




                          Black king is in illegal double check, could not be discovered by legal movement of either offending piece




                          7.




                          Black rook starting h8 could not have legally escaped, due to number of remaining black pawns, absence of rook considered illegal




                          8.




                          White pawns are at least 3 places off home columns, black still has 16 pieces, though missing a knight, at least two illegal places off home




                          9.




                          Pawn f7 is illegal, cannot have captured the required 2 pieces to return to home column behind black pawn as black not missing enough pieces




                          10.




                          Even assuming pawn g3 to be the interloper, black pawns are a minimum of 4 places off home column, white is missing 4 pieces, but rook a1 could only have been captured by a knight or queen, at least one illegal place off home




                          Not illegal but might look it



                          1.




                          Bishop f3 check could be discovered by movement of knight h4 from g2, otherwise it's really hard to get into that position





                          1. This ties into illegal #10



                          Assuming bishop b1 to have been placed there at the start of the game, white rook starting a1 could not have legally escaped, but could have been captured by a knight or queen. Black bishop could not have been used.







                          share|improve this answer




























                            3














                            Definitely illegal



                            1.




                            Black has 9 pawns




                            2.




                            Rook a1 could be legal in isolation as a promoted pawn, but in light of #1, I'm declaring it illegal




                            3.




                            Bishop b1 cannot get there




                            4.




                            Both white bishops are on white, white has 8 pawns, one bishop is illegal




                            5.




                            Both kings are in check




                            6.




                            Black king is in illegal double check, could not be discovered by legal movement of either offending piece




                            7.




                            Black rook starting h8 could not have legally escaped, due to number of remaining black pawns, absence of rook considered illegal




                            8.




                            White pawns are at least 3 places off home columns, black still has 16 pieces, though missing a knight, at least two illegal places off home




                            9.




                            Pawn f7 is illegal, cannot have captured the required 2 pieces to return to home column behind black pawn as black not missing enough pieces




                            10.




                            Even assuming pawn g3 to be the interloper, black pawns are a minimum of 4 places off home column, white is missing 4 pieces, but rook a1 could only have been captured by a knight or queen, at least one illegal place off home




                            Not illegal but might look it



                            1.




                            Bishop f3 check could be discovered by movement of knight h4 from g2, otherwise it's really hard to get into that position





                            1. This ties into illegal #10



                            Assuming bishop b1 to have been placed there at the start of the game, white rook starting a1 could not have legally escaped, but could have been captured by a knight or queen. Black bishop could not have been used.







                            share|improve this answer


























                              3












                              3








                              3






                              Definitely illegal



                              1.




                              Black has 9 pawns




                              2.




                              Rook a1 could be legal in isolation as a promoted pawn, but in light of #1, I'm declaring it illegal




                              3.




                              Bishop b1 cannot get there




                              4.




                              Both white bishops are on white, white has 8 pawns, one bishop is illegal




                              5.




                              Both kings are in check




                              6.




                              Black king is in illegal double check, could not be discovered by legal movement of either offending piece




                              7.




                              Black rook starting h8 could not have legally escaped, due to number of remaining black pawns, absence of rook considered illegal




                              8.




                              White pawns are at least 3 places off home columns, black still has 16 pieces, though missing a knight, at least two illegal places off home




                              9.




                              Pawn f7 is illegal, cannot have captured the required 2 pieces to return to home column behind black pawn as black not missing enough pieces




                              10.




                              Even assuming pawn g3 to be the interloper, black pawns are a minimum of 4 places off home column, white is missing 4 pieces, but rook a1 could only have been captured by a knight or queen, at least one illegal place off home




                              Not illegal but might look it



                              1.




                              Bishop f3 check could be discovered by movement of knight h4 from g2, otherwise it's really hard to get into that position





                              1. This ties into illegal #10



                              Assuming bishop b1 to have been placed there at the start of the game, white rook starting a1 could not have legally escaped, but could have been captured by a knight or queen. Black bishop could not have been used.







                              share|improve this answer














                              Definitely illegal



                              1.




                              Black has 9 pawns




                              2.




                              Rook a1 could be legal in isolation as a promoted pawn, but in light of #1, I'm declaring it illegal




                              3.




                              Bishop b1 cannot get there




                              4.




                              Both white bishops are on white, white has 8 pawns, one bishop is illegal




                              5.




                              Both kings are in check




                              6.




                              Black king is in illegal double check, could not be discovered by legal movement of either offending piece




                              7.




                              Black rook starting h8 could not have legally escaped, due to number of remaining black pawns, absence of rook considered illegal




                              8.




                              White pawns are at least 3 places off home columns, black still has 16 pieces, though missing a knight, at least two illegal places off home




                              9.




                              Pawn f7 is illegal, cannot have captured the required 2 pieces to return to home column behind black pawn as black not missing enough pieces




                              10.




                              Even assuming pawn g3 to be the interloper, black pawns are a minimum of 4 places off home column, white is missing 4 pieces, but rook a1 could only have been captured by a knight or queen, at least one illegal place off home




                              Not illegal but might look it



                              1.




                              Bishop f3 check could be discovered by movement of knight h4 from g2, otherwise it's really hard to get into that position





                              1. This ties into illegal #10



                              Assuming bishop b1 to have been placed there at the start of the game, white rook starting a1 could not have legally escaped, but could have been captured by a knight or queen. Black bishop could not have been used.








                              share|improve this answer














                              share|improve this answer



                              share|improve this answer








                              edited 9 hours ago

























                              answered 9 hours ago









                              Separatrix

                              27116




                              27116























                                  2















                                  Bb1, no legal way to reach there
                                  Ra1, no legal way to reach there, due to Bb1
                                  Kh1, in check, other King in check
                                  Kc8, in double check, other King in check
                                  f7, no legal way to reach there
                                  Bf8, no legal way to reach there
                                  2 white bishops, with 8 pawns. Does this count as two illegals?







                                  share|improve this answer








                                  New contributor




                                  Kashyap is a new contributor to this site. Take care in asking for clarification, commenting, and answering.
                                  Check out our Code of Conduct.























                                    2















                                    Bb1, no legal way to reach there
                                    Ra1, no legal way to reach there, due to Bb1
                                    Kh1, in check, other King in check
                                    Kc8, in double check, other King in check
                                    f7, no legal way to reach there
                                    Bf8, no legal way to reach there
                                    2 white bishops, with 8 pawns. Does this count as two illegals?







                                    share|improve this answer








                                    New contributor




                                    Kashyap is a new contributor to this site. Take care in asking for clarification, commenting, and answering.
                                    Check out our Code of Conduct.





















                                      2












                                      2








                                      2







                                      Bb1, no legal way to reach there
                                      Ra1, no legal way to reach there, due to Bb1
                                      Kh1, in check, other King in check
                                      Kc8, in double check, other King in check
                                      f7, no legal way to reach there
                                      Bf8, no legal way to reach there
                                      2 white bishops, with 8 pawns. Does this count as two illegals?







                                      share|improve this answer








                                      New contributor




                                      Kashyap is a new contributor to this site. Take care in asking for clarification, commenting, and answering.
                                      Check out our Code of Conduct.










                                      Bb1, no legal way to reach there
                                      Ra1, no legal way to reach there, due to Bb1
                                      Kh1, in check, other King in check
                                      Kc8, in double check, other King in check
                                      f7, no legal way to reach there
                                      Bf8, no legal way to reach there
                                      2 white bishops, with 8 pawns. Does this count as two illegals?








                                      share|improve this answer








                                      New contributor




                                      Kashyap is a new contributor to this site. Take care in asking for clarification, commenting, and answering.
                                      Check out our Code of Conduct.









                                      share|improve this answer



                                      share|improve this answer






                                      New contributor




                                      Kashyap is a new contributor to this site. Take care in asking for clarification, commenting, and answering.
                                      Check out our Code of Conduct.









                                      answered 5 hours ago









                                      Kashyap

                                      1213




                                      1213




                                      New contributor




                                      Kashyap is a new contributor to this site. Take care in asking for clarification, commenting, and answering.
                                      Check out our Code of Conduct.





                                      New contributor





                                      Kashyap is a new contributor to this site. Take care in asking for clarification, commenting, and answering.
                                      Check out our Code of Conduct.






                                      Kashyap is a new contributor to this site. Take care in asking for clarification, commenting, and answering.
                                      Check out our Code of Conduct.






























                                          draft saved

                                          draft discarded




















































                                          Thanks for contributing an answer to Puzzling Stack Exchange!


                                          • Please be sure to answer the question. Provide details and share your research!

                                          But avoid



                                          • Asking for help, clarification, or responding to other answers.

                                          • Making statements based on opinion; back them up with references or personal experience.


                                          Use MathJax to format equations. MathJax reference.


                                          To learn more, see our tips on writing great answers.





                                          Some of your past answers have not been well-received, and you're in danger of being blocked from answering.


                                          Please pay close attention to the following guidance:


                                          • Please be sure to answer the question. Provide details and share your research!

                                          But avoid



                                          • Asking for help, clarification, or responding to other answers.

                                          • Making statements based on opinion; back them up with references or personal experience.


                                          To learn more, see our tips on writing great answers.




                                          draft saved


                                          draft discarded














                                          StackExchange.ready(
                                          function () {
                                          StackExchange.openid.initPostLogin('.new-post-login', 'https%3a%2f%2fpuzzling.stackexchange.com%2fquestions%2f78081%2fa-very-illegal-position%23new-answer', 'question_page');
                                          }
                                          );

                                          Post as a guest















                                          Required, but never shown





















































                                          Required, but never shown














                                          Required, but never shown












                                          Required, but never shown







                                          Required, but never shown

































                                          Required, but never shown














                                          Required, but never shown












                                          Required, but never shown







                                          Required, but never shown







                                          Popular posts from this blog

                                          An IMO inspired problem

                                          Management

                                          Investment